Argument Structure Questions - - Question 4
Of every 100 burglar alarms police answer, 99 are false alarms. This situation causes an enormous and dangerous drain...
Replies
Mehran November 13, 2017
Hi @lizaclark95, thanks for your post. This is an Argument Structure question. You are asked to identify the role played by the statement that burglar alarm systems, unlike care alarm systems, are effective in deterring burglaries.Let's review the stimulus carefully. The highlighted statement comes right before "so the only acceptable solution is to fine burglar alarm system owners the cost of 45 minutes of police time for each false alarm their systems generate."
According to the author of this stimulus, therefore, the key fact that informs "the only acceptable solution" is that "burglar alarm systems, unlike car alarm systems, are effective in deterring burglaries . . ."
Thus, we can select answer choice (C): this statement "provides a basis for excluding as unacceptable one obvious alternative to the proposal of fining owners of burglar alarm systems for false alarms." The obvious alternative is that all alarm owners be fined - not just burglar alarm owners.
Hope this helps! Please let us know if you have additional questions.
LadyMae December 30, 2018
Ugh I would object to this question if I didn't know the people making the tests are deliberately trying to confuse us. They achieved their goal well here.Any idea how to better see such "obvious alternatives?" I didn't get that AT ALL from reading the passage.
Ravi December 31, 2018
@LadyMae,Happy to help! This is a very tricky question, and you're right—the
test writers are trying to mess with test takers on this one.
You asked about better ideas for how to see such "obvious
alternatives." Other than closely reading the stimulus and pausing
after each sentence to see how what you just read can be applied to
the previous sentence(s), there's not a great way to do this. However,
the good news is that you actually don't have to be able to see such
"obvious alternatives" when reading the stimulus to get this answer
correct. As long as you have a firm grasp of the argument structure
and what the conclusion of the stimulus is, you can answer this
question correctly without knowing what the obvious alternative is.
The conclusion of the argument is that the only acceptable solution is
to fine burglar alarm system owners the cost of 45 minutes of police
time for each false alarm their systems generate. This is supported by
the premise that burglar alarm systems, unlike car alarm systems, are
effective in deterring burglaries.
The question stem asks us to identify the role what the statement that
burglar alarm systems, unlike car alarm systems, plays in the
argument. Based on our analysis, we know that this statement is a
premise that is supporting the overall conclusion of the argument.
Therefore, when we read the answer choices, we're looking for an
answer that matches our prediction: this statement is a premise
supporting the conclusion.
Answer A is incorrect because there is nowhere in the stimulus where
restrictions on owners of burglar alarms vs. owners of car alarms is
discussed. It's says it "justifies," which is what a premise does, but
the rest of answer A is nonsense. We can get rid of this.
Answer B is incorrect because it is not providing background
information needed for the statement that the number of burglar alarms
police are called on to answer is great enough to be a drain on public
resources. This statement is provided as background information (a
fact) in the beginning of the argument. We can eliminate this answer.
Answer C is correct because it provides a great paraphrase of what
we're looking for. It says that it provides a basis for excluding as
unacceptable one obvious alternative to the proposal of finding owners
of burglar alarm systems for false alarms. "Providing a basis" means
it's providing support, so we know answer C is describing the
statement as a premise. The proposal of fining owners of burglar alarm
systems for false claims is the conclusion that we identified, so this
answer is saying that it's supporting that conclusion. Even if the
part of answer C that says "excluding as unacceptable one obvious
alternative" is something we don't catch immediately, the rest of the
answer is strong. On our first pass through the answer choices, we may
not circle this answer immediately, but after looking at D and E and
how bad they are, we'll be able to select this answer choice
confidently.
Answer D is incorrect because the argument deals with imposing fines
as the only acceptable solution to the problem of false alarms. The
argument is not about police having a greater inclination to respond
to burglar alarms than to car alarms. The argument is not about car
alarms at all, and we could remove the bit about car alarms from the
argument without affecting it. We can get rid of this answer.
Answer E is incorrect because the purported statement that E is saying
is being explained is something that is not mentioned in the argument.
We can eliminate this answer.
The only decent answer we have left is C. As noted above, we know that
it's a premise supporting the conclusion, and C is the only answer
that remotely resembles that.
Now, let's look at how C is ruling out one obvious alternative to the
proposal of fining owners of burglar alarm systems for false alarms.
The purpose of the statement that burglar alarms, unlike car alarm
systems, are effective in deterring burglaries is used as a way of
excluding the possibility of just getting rid of alarms. It shows that
they have some utility, and it supports the contention that the only
acceptable solution si to fine burglar alarm system owners the cost of
45 minutes of police time for each false alarm their systems generate.
Does this make sense? Let us know if you have any more questions!
Ashley123 November 20, 2021
@Mehran , it seems like some of the answers here are conflicting and it's really confusing.First, I thought we were supposed to choose answers that must be true (meaning they have to be stated in the stimulus and we can't bring in outside information). Why does this question not follow that rule?
Also, I read the statement to mean that the author was suddenly mentioning car alarms because he or she was comparing them to the burglar alarms. I thought that stating the burglar alarms are effective but the car alarms are not was supposed to support the conclusion because the conclusion only mentions fining the burglar alarms and not the car alarms (which makes sense because the burglar alarms deter burglars so police are wasting time by responding to them but car alarms don't deter burglars so police wouldn't be wasting time by responding to those 'legitimate calls').
So wouldn't the statement be justifying why the conclusion only mentions fining burglar alarms but not fining car alarms too? I thought answer A was basically just restating the conclusion. Especially since it is actually stated in the stimulus and would be a more obvious solution to fine all alarms, why are we asked to assume another more extreme solution (that is not even mentioned) is being rejected. Why else would it even mention car alarms if it is not drawing a comparison between the two? I'm so confused by this and keep ending up with different reasonings that lead me back to answer A being supported. Every time I try to make sense of answer C, it just leads me back to A because I can't understand why it would even mention car alarms if it is just trying to show that burglar alarms are effective so we need to keep them but they waste police time so they need to be fined for it.
Please help! It's terribly confusing and I've been going in circles trying to rationalize it. I just want to make sure I get all of these answers right so I can have a solid foundation and score like all of you did before you went to law school.
Thank you,
A very tired prospective law student
Ravi February 5, 2022
@Ashley123, A doesn't work because of the phrase "owners of car alarms." The author never discusses whether we should place restrictions on car alarms. It's possible she thinks they should be banned.C works because "a basis" means the same thing as a premise. The phrase in question is providing support in this argument. And even though it's not explicitly mentioned, the "obvious alternative" would be to ban burglar alarms. The conclusion is that we should not do that; the only thing we should do is charge for false alarms.